Radiology Faculty--Answering Questions/"AMA"

This forum made possible through the generous support of SDN members, donors, and sponsors. Thank you.

RadiologyPD

Full Member
7+ Year Member
Joined
Mar 4, 2017
Messages
286
Reaction score
626
Hello, I am a Radiology PD who was asked by an SDN administrator who is friend of one of my former fellows to participate in this thread. My understanding is that there may be at least one other PD (by the handle of "Radiology_Advisor") who will participate for now. Here is the request I received:

We are looking to increase attending presence on the forums especially in those specialties, such as DR, where students may not get a lot of exposure during the premed and med school years. Our "AMA" (Ask Me Anything") threads are immensely popular and of course, having PD input benefits all users as your experience and insight is something that we feel would be invaluable and not easily accessible elsewhere. Would you be interested in participating?

For now I prefer to stay anonymous, but it may help to know a bit about me. I graduated from a big Midwest state medical school almost 30 years ago and did my internship/residency out West. I did not entertain the idea of becoming a radiologist until my M3 year, after doing an elective in Radiology that I chose in order to be more familiar with how to get films/reports on my patients prior to my subsequent Internal Medicine and Surgery rotations (in the old days, students were judged by how good they were at getting what you needed from the radiology records department before rounds!). A big part of the reason I chose radiology is because I enjoyed the one-to-one teaching interactions that I saw the academic radiologists doing more than what I saw the academic internists and academic surgeons doing, and I wanted to stay in academics if possible. I've only been at two academic centers in my career--staying at the institution I did my residency & fellowship as an attending for about 5 years, then moving to my present position, which is also out West. I was a Chief Resident and on the residency selection committee at my former institution, and have been a fellowship program director and more recently am the residency program director at my current institution. As a result, I've been involved with the training of many radiologists.

It might have made more sense to start this thread after the upcoming Match, as I'm sure this forum will be buzzing with the results of the Match for a while, but I'm forging ahead now as requested. Heads up, I'm not going to engage into conversations about which program is better, or "chance me" requests, other than to perhaps direct those sorts of inquiries into more a more general discussion of underlying principles. Nevertheless, feel free to ask whatever, and if I don't particularly feel like I should engage or if I don't have a good answer, I won't hesitate to say so. Also, I may not be able to be as responsive as others, but I'll do my best.

Ok, that's it for now. Good luck to all of you who are awaiting the upcoming Match results!

Members don't see this ad.
 
  • Like
Reactions: 18 users
Thanks so much for doing this.

What are PD's generally looking for in a residency application? What can set an applicant apart?

What should applicants be looking for in a residency?

Does the choice (medicine vs surgery vs transitional) and location of internship have any significance?

Do DR PD's look down on applications from IR/DR applicants? Is there any disadvantage to dual applying?
 
1. What are some things you see residents coming to learn toward the end of residency they wish they had known when they first started out residency training?

2. Does independent call during residency really make for better residents and overall better radiologist? What have you found to be the ideal call set up?

3. Job market is currently improving, but still has a way to go. How does the job market affect the dynamic of the residency program, if at all?

4. Do resi fellowships enhance the overall learning experience for residents? Have there been unforeseen consequences of Resi-fellowships?

5. What do you feel are the top 5 traits that make for a successful radiology resident?
 
Members don't see this ad :)
I am the person @RadiologyPD was referring to.

I've been an academic attending radiologist for over 25 years and love my job. I have been the director for our medical student radiology rotation and the faculty advisor for medical students interested in radiology for over 10 years.

If you have questions about applying to radiology - or about radiology in general - please feel free to ask.
 
  • Like
Reactions: 14 users
Welcome to SDN!

We appreciate your time and future input; most of us who have stayed on through medical school, residency and are now in practice find it very rewarding to teach students and enjoy our time here.

So that our members are aware, neither @RadiologyPD or @Radiology_Advisor are paid SDN staff members; they have full time practices and are participating out of kindness and a desire to educate. Please be patient when waiting for answers and don't overwhelm them. Its a honor to have them here!
 
  • Like
Reactions: 10 users
What should applicants be looking for in a residency?

- Depending your preferences - kinds of clinical cases (number and variety), teaching (didactic and day-to-day), researching opportunities, teaching opportunities (e.g., to medical students), voice in decision-making

Does the choice (medicine vs surgery vs transitional) and location of internship have any significance?

Choice of internship
- Surgery for IR generally
- Medicine or surgery depending on your preference. Each has advantages - surgery helps you visualize the anatomy better; medicine helps you understand disease pathology better. At the end of radiology residency, studies have shown no difference. If undecided, go for surgery as it will make your first year of radiology easier.
- Other - e.g., pediatrics - if you want to go into a specific specialty such as peds radiology.
- Transitional - my feeling is that this is like a 5th year of med school and you will not be able to anyone's senior at the end of the year so you will have less confidence as a physician

Location
- Nice to be near your radiology program so you won't have to move between internship and residency
- Or... you can live for a year in a place you always wanted to live in...
 
  • Like
Reactions: 8 users
how much importance do you place on radiology letters of recommendation? would you recommend getting a letter from the biggest name possible, or would a small community program director be as good if they wrote a good letter?

also, do you recommend away rotations? particularly if your home program is a small, community one?

thanks for doing this!!
 
These are great questions. I'll try to tackle them in depth, but it's going to take a while for me.

I thought it might help to understand process for selection for interview at my place. You can glean what I'm looking for at this stage of the application process--it only applies to selection for interview, not ranking after interview.

Here's what I do:

1. I download the ERAS application data into an excel spreadsheet which allows me to create custom parameters and is easier for me to filter/sort and quickly review. ERAS allows you to download certain parameters. Each applicant is a row, and I type notes and create formulas into custom columns that I create.

2. I personally select the resident applicants that we will interview from the hundreds of applications we get. It’s just too slow and hard to do this with a committee. At my old institution, this was also done by essentially 1 person (not me back then), but I don’t know if they have changed that. I have a “2nd reviewer” for borderline cases, maybe I use that for 3-4 applications each year.

3. If an applicant has taken USMLE Step 2, I average that score with Step 1. If an applicant hasn’t’ taken USMLE Step 2, I add 5 points to their Step 1 score and create a “derived Step 2” score (edit 3/2018--for the 2018 Match, we just used the Step 1 score as the person's Step 2 score for our spreadsheet/formulas--this really does underestimate the Step 2 score, since most people do better on Step 2 than Step 1). In our applicant pool, the average applicant has a Step 2 score that is about 7-10 points higher than Step 1. So it hurts you a little if you haven’t taken Step 2 because my assumed score for you is not as high as it statistically would be if you are the average applicant.

4. Turns out I apply a formula to the “combined USMLE score” that discounts super-high achievement. Essentially, as your score on Step 1 or Step 2 gets higher from 250, you get less added points. For example, someone who gets 250 on Step 1 and Step 2 has a combined score of 500 in my system. However, someone who gets 265 on Step 1 and Step 2 has a combined score of 515 in my system, not 530 (and the score is capped there, meaning even higher scores don't add points). I don’t want super-high achievement on USMLE to dominate an applicant’s eventual “overall score”, or to make up for lower clinical grades and interview scores. This last year, in our program, our applicants had a mean combined USMLE score of 494, and a median of 498, with a standard deviation of 19. Remember, those scores have been adjusted to discount performance substantially above 250 in a formulaic way that progressively discounts numbers the farther above 250.

5. We filter out the applications based on USMLE scores, but we use a really low threshold—in our case, we use a “soft” 220 USMLE step 1 score, which generally means I will consider applicants in the 215-220 range based on strength of school and how well they did on step 2, as well as other factors. Right now the average Step 1 score for all medical students is about 228 or so, I believe, so allowing someone to be as low as 215 in our case means you don’t necessarily have to be book smart to get into our program. However, radiology boards is now a computerized test, and we don’t want to worry about residents who may become outstanding radiologists but who might struggle with tests. There is too much of a penalty for our program in future applicant perception if one of our residents fails the boards. (edit 3/2018: despite our willingness to look at applicants with lower than average board scores, the average Step 1 score for the applicants we matched in March 2018 was 250, and the average Step 2 score was 258).

6. Our residency essentially filters out applicants who are IMGs—to be honest, I think there are some outstanding candidates in this group particularly from those individuals who are not from the US, but the problem is that it is really difficult to find those for me because we don’t use test scores as much in our ranking process. Communication skills are very important to us, and that can be a sticking point for some IMGs who did not grow up in the US that we don’t understand until the interview, and I don’t want to waste people’s time with interviews if there is a low chance of success. On the other hand, we understand there are some life circumstances and other legitimate reasons why some applicants who are US based ended up doing medical school internationally. So I download these into my spreadsheet, dig deeper at maybe the candidates with board scores above 250 to see if I recognize the school, review publications, special experiences, special circumstances, etc. Sometimes a colleague will ask me to look carefully at a person that someone in their field has highlighted for them. We do interview a few IMGs every year (< 5), and some are quite good. However, they face hurdles all along the way in our ranking process—just being honest.

7. We do the same as #6 for DO candidates. We do interview a few every year (< 3), but we believe there is a penalty for our program in future applicants if we have a number of DO residents, mainly because there is the perception that we couldn’t attract the best MD candidates. It’s unfortunate for some DO students who are going to be great, but it is reality.

8. One of the filters I use to select who to interview is 3rd year core clerkship performance. This is a really tough metric, since the schools are all over the place. Believe it or not, I spend the time to create a “translation formula” that looks at the % of Honors/High Pass/Pass from each US medical school from which we receive an application, so I can compare the performance of students from different schools—even then, it’s not easy and likely not accurate. For example, just looking at my spreadsheet for this year, I see that for the core clerkships that we review, at the University of Central Florida 52% of students got Honors and 48% got High Pass (no one just passed), whereas at Florida International University 14% got Honors and 29% got High Pass. I have a convoluted formula that tries to “normalize” this data, so that the student at FIU that got High Pass is given the same number of points as the student at UCF that got Honors.

9. Since I have to actually open up the application to get the 3rd year clerkship performance and % honors/high pass/pass data, I do quickly jot down a few notes about the candidate at this time—I’ll jot down a few sentences about the candidate about their particular timeline, skimming the personal statement, looking quickly at the research history, etc. I quickly look at the Dean’s letter (if available) to look for red flags (repeat courses). I do put down what “quartile” the Dean’s letter says you are in. I don’t have time to review LORs at this point UNLESS I can tell that the candidate is probably going to be on my “borderline” for selecting for interview. For example, here is a typical fictional set of notes that I might jot down in my “comment” box for a candidate at this point: “Brown undergrad; 1 yr gap spent as research intern for startup and also doing volunteer work; PS specifically mentions us”. In the “Dean’s letter” box, I might put “2nd quartile”.

10. My spreadsheet combines the “3rd year clerkship score” with the USMLE average score (either real or derived) in a way that weights the clerkship score. This gives each applicant a “non-interview” score—that is, their score without consideration of the interview. As you will see later, the eventual evaluation of a candidate relies more on the interview than this score. But this is the metric that helps us decide who to interview.

11. I sort my spreadsheet using the “non-interview” score to decide who to interview. For about 67% of our interview slots, I just take them from the top. For the final 33%, I use a different lower threshold for interviewing applicants who are considered “local” (from our med school and schools within about 100 miles of our urban program), “regional” (about 500 miles), and “national” (everyone else). The reason we do this is because we find that applicants are more likely to not cancel interviews and match with us if they are local or regional. We also don’t want to piss off our school/the local schools and not interview their students—to a degree. We won’t interview if someone is really not going to be up to snuff based on performance. If a national candidate has ties to our area that are obvious in the application (I look at permanent address and undergraduate location), then they get put in the local pile. Similarly, if the applicant did a rotation with us, we consider them in the local pile even if they aren’t. However, we don’t take that many medical students outside our own medical school for rotations. Along with the varying thresholds based on geography, I look at my comments and the “Dean’s letter comments” to decide who to select for these final 33%.

12. If the applicant is AOA, they almost always get an interview. Turns out they always score above my threshold on the non-interview score anyway (makes sense, since AOA status is generally a function of traits that are well reflected in the USMLE scores and core clerkship grades). However, we sometimes have an AOA student who I end up not interviewing, because of something in the application that is a red flag that I can easily see from my spreadsheet (repeating a course, something in the Dean’s letter).

13. We slightly “overinterview” in our program—basically, interview about 10-14 applicants for every spot, even though we typically fill our spots at the 5-8 applicant/spot filled mark—and even then, about 3/4ths of our class is filled before the 5 applicant/spot mark. The reason we do this is because we don’t trust that our combined “USMLE + clinical clerkship” score is so precise that we can rely on it, and we sometimes find that applicants we end up ranking fairly high would not have been offered an interview with us if we had not “overinterviewed”. (note: in 2018, we ended up filling at the 4 applicant/spot mark, so we are reducing the number of people we interview).

So, a few things should be evident so far, regarding “how to get an interview” at my program:

1. Do well on USMLE tests, but no need to ace it. Does it help to get a 270 vs. a 250?—not really.

2. Do well on 3rd year core clerkships.

3. Be local or communicate your connection to my community in some way—it lowers the threshold for you getting an interview. Say it in the personal statement if it is true. Even then, you might want to email me in advance if that local connection isn’t obvious in the application.

4. I don’t have time to consider whether you decided not to do a rotation with us at the “select for interview” stage. I don’t have time or an easy way to consider the strength of your research record. I don’t have time to look at your Dean’s letter in depth beyond just trying to make sure there is no coded “red flag” and to understand your relative performance. I don’t have time to consider your extracurricular activities.

Once you get selected for interview, the selection metrics become more nuanced—another long discussion for another day.
 
Last edited:
  • Like
  • Hmm
  • Love
Reactions: 92 users
These are great questions. I'll try to tackle them in depth, but it's going to take a while for me.

I thought it might help to understand process for selection for interview at my place. You can glean what I'm looking for at this stage of the application process--it only applies to selection for interview, not ranking after interview.

Here's what I do:

1. I download the ERAS application data into an excel spreadsheet which allows me to create custom parameters and is easier for me to filter/sort and quickly review. ERAS allows you to download certain parameters. Each applicant is a row, and I type notes and create formulas into custom columns that I create.

2. I personally select the resident applicants that we will interview from the hundreds of applications we get. It’s just too slow and hard to do this with a committee. At my old institution, this was also done by essentially 1 person (not me back then), but I don’t know if they have changed that. I have a “2nd reviewer” for borderline cases, maybe I use that for 3-4 applications each year.

3. If an applicant has taken USMLE Step 2, I average that score with Step 1. If an applicant hasn’t’ taken USMLE Step 2, I add 5 points to their Step 1 score and create a “derived Step 2” score. In our applicant pool, the average applicant has a Step 2 score that is about 7-10 points higher than Step 1. So it hurts you a little if you haven’t taken Step 2 because my assumed score for you is not as high as it statistically would be if you are the average applicant.

4. Turns out I apply a formula to the “combined USMLE score” that discounts super-high achievement. Essentially, as your score on Step 1 or Step 2 gets higher from 250, you get less added points. For example, someone who gets 250 on Step 1 and Step 2 has a combined score of 500 in my system. However, someone who gets 265 on Step 1 and Step 2 has a combined score of 515 in my system, not 530. I don’t want super-high achievement on USMLE to dominate an applicant’s eventual “overall score”, or to make up for lower clinical grades and interview scores. This last year, in our program, our applicants had a mean combined USMLE score of 494, and a median of 498, with a standard deviation of 19. Remember, those scores have been adjusted to discount performance substantially above 250 in a formulaic way that progressively discounts numbers the farther above 250.

5. We filter out the applications based on USMLE scores, but we use a really low threshold—in our case, we use a “soft” 220 USMLE step 1 score, which generally means I will consider applicants in the 215-220 range based on strength of school and how well they did on step 2, as well as other factors. Right now the average Step 1 score for all medical students is about 228 or so, I believe, so allowing someone to be as low as 215 in our case means you don’t necessarily have to be book smart to get into our program. However, radiology boards is now a computerized test, and we don’t want to worry about residents who may become outstanding radiologists but who might struggle with tests. There is too much of a penalty for our program in future applicant perception if one of our residents fails the boards.

6. Our residency essentially filters out applicants who are IMGs—to be honest, I think there are some outstanding candidates in this group particularly from those individuals who are not from the US, but the problem is that it is really difficult to find those for me because we don’t use test scores as much in our ranking process. Communication skills are very important to us, and that can be a sticking point for some IMGs who did not grow up in the US that we don’t understand until the interview, and I don’t want to waste people’s time with interviews if there is a low chance of success. On the other hand, we understand there are some life circumstances and other legitimate reasons why some applicants who are US based ended up doing medical school internationally. So I download these into my spreadsheet, dig deeper at maybe the candidates with board scores above 250 to see if I recognize the school, review publications, special experiences, special circumstances, etc. Sometimes a colleague will ask me to look carefully at a person that someone in their field has highlighted for them. We do interview a few IMGs every year (< 5), and some are quite good. However, they face hurdles all along the way in our ranking process—just being honest.

7. We do the same as #3 for DO candidates. We do interview a few every year (< 3), but we believe there is a penalty for our program in future applicants if we have a number of DO residents, mainly because there is the perception that we couldn’t attract the best MD candidates. It’s unfortunate for some DO students who are going to be great, but it is reality.

8. One of the filters I use to select who to interview is 3rd year core clerkship performance. This is a really tough metric, since the schools are all over the place. Believe it or not, I spend the time to create a “translation formula” that looks at the % of Honors/High Pass/Pass from each US medical school from which we receive an application, so I can compare the performance of students from different schools—even then, it’s not easy and likely not accurate. For example, just looking at my spreadsheet for this year, I see that for the core clerkships that we review, at the University of Central Florida 52% of students got Honors and 48% got High Pass (no one just passed), whereas at Florida International University 14% got Honors and 29% got High Pass. I have a convoluted formula that tries to “normalize” this data, so that the student at FIU that got High Pass is given the same number of points as the student at UCF that got Honors.

9. Since I have to actually open up the application to get the 3rd year clerkship performance and % honors/high pass/pass data, I do quickly jot down a few notes about the candidate at this time—I’ll jot down a few sentences about the candidate about their particular timeline, skimming the personal statement, looking quickly at the research history, etc. I quickly look at the Dean’s letter (if available) to look for red flags (repeat courses). I do put down what “quartile” the Dean’s letter says you are in. I don’t have time to review LORs at this point UNLESS I can tell that the candidate is probably going to be on my “borderline” for selecting for interview. For example, here is a typical fictional set of notes that I might jot down in my “comment” box for a candidate at this point: “Brown undergrad; 1 yr gap spent as research intern for startup and also doing volunteer work; PS specifically mentions us”. In the “Dean’s letter” box, I might put “2nd quartile”.

10. My spreadsheet combines the “3rd year clerkship score” with the USMLE average score (either real or derived) in a way that weights the clerkship score. This gives each applicant a “non-interview” score—that is, their score without consideration of the interview. As you will see later, the eventual evaluation of a candidate relies more on the interview than this score. But this is the metric that helps us decide who to interview.

11. I sort my spreadsheet using the “non-interview” score to decide who to interview. For about 67% of our interview slots, I just take them from the top. For the final 33%, I use a different lower threshold for interviewing applicants who are considered “local” (from our med school and schools within about 100 miles of our urban program), “regional” (about 500 miles), and “national” (everyone else). The reason we do this is because we find that applicants are more likely to not cancel interviews and match with us if they are local or regional. We also don’t want to piss off our school/the local schools and not interview their students—to a degree. We won’t interview if someone is really not going to be up to snuff based on performance. If a national candidate has ties to our area that are obvious in the application (I look at permanent address and undergraduate location), then they get put in the local pile. Similarly, if the applicant did a rotation with us, we consider them in the local pile even if they aren’t. However, we don’t take that many medical students outside our own medical school for rotations. Along with the varying thresholds based on geography, I look at my comments and the “Dean’s letter comments” to decide who to select for these final 33%.

12. If the applicant is AOA, they almost always get an interview. Turns out they always score above my threshold on the non-interview score anyway (makes sense, since AOA status is generally a function of traits that are well reflected in the USMLE scores and core clerkship grades). However, we sometimes have an AOA student who I end up not interviewing, because of something in the application that is a red flag that I can easily see from my spreadsheet (repeating a course, something in the Dean’s letter).

13. We typically “overinterview” in our program—basically, interview about 15-20 applicants for every spot, even though we typically fill our spots at the 7-10 applicant/spot filled mark—and even then, about 3/4ths of our class is filled at the 5 applicant/spot mark. The reason we do this is because we don’t trust that our combined “USMLE + clinical clerkship” score is so precise that we can rely on it, and we sometimes find that applicants we end up ranking fairly high would not have been offered an interview with us if we had not “overinterviewed”.

So, a few things should be evident so far, regarding “how to get an interview” at my program:

1. Do well on USMLE tests, but no need to ace it. Does it help to get a 270 vs. a 250?—not really.

2. Do well on 3rd year core clerkships.

3. Be local or communicate your connection to my community in some way—it lowers the threshold for you getting an interview. Say it in the personal statement if it is true. Even then, you might want to email me in advance if that local connection isn’t obvious in the application.

4. I don’t have time to consider whether you decided not to do a rotation with us at the “select for interview” stage. I don’t have time or an easy way to consider the strength of your research record. I don’t have time to look at your Dean’s letter in depth beyond just trying to make sure there is no coded “red flag” and to understand your relative performance. I don’t have time to consider your extracurricular activities.

Once you get selected for interview, the selection metrics become more nuanced—another long discussion for another day.

This is the most helpful post ive ever seen
 
  • Like
Reactions: 22 users
This is the most helpful post ive ever seen

Agreed.

I think admin. is taking huge strides to make up for removing our ROL thread, and the result is spectacular. Organizing and getting the ball rolling on something like this is a significant undertaking, not to mention the commitment by two PDs to the forum.

I can't say I've come across other sites that would go to this length to make things right and further enhance the forum.

Hats off to ya, SDN.
 
Last edited:
  • Like
Reactions: 3 users
Thank you for taking the time to do this, Radiology_Advisor and RadiologyPD, awesome responses so far.

1. As an average applicant planning to apply broadly to programs across the country, can you speak to the pros and cons of attending a community program (instead of a university program), in particular for fellowship prospects or going into academics thereafter? It is common for many community programs to claim that their residents match into great fellowships--how much stock should we put into this?

2. I am interested in DR currently. With the opening of IR/DR residencies, how do you think the competitiveness for DR residency spots will change in light of this?
 

Do DR PD's look down on applications from IR/DR applicants? Is there any disadvantage to dual applying?

I am interested in DR currently. With the opening of IR/DR residencies, how do you think the competitiveness for DR residency spots will change in light of this?

For now, you should dual apply - over the next few years, as the programs are more separated and we know how competitive each is, the answer might be more nuanced.​

As an average applicant planning to apply broadly to programs across the country, can you speak to the pros and cons of attending a community program (instead of a university program), in particular for fellowship prospects or going into academics thereafter? It is common for many community programs to claim that their residents match into great fellowships--how much stock should we put into this?
  • Radiology learning, even more than in other fields in medicine, is based on your own effort and reading. Most community programs will have enough cases of most kinds of cases.
  • A university program may offer more opportunity for research and likely more exposure to state-of-the-art protocols and equipment.
  • Community programs definitely do match their residents into great fellowships. We have an attending who started at a community program and went to a great fellowship, and is now one of the most popular attendings, winning Teach of the Year and getting substantial grants. The fellowships will pick the good resident from a community program over an average resident from a university program. Think 4 year ivy league vs. community college/transfer to 4 year to get into med school. May help, but not everything.

How much importance do you place on radiology letters of recommendation? would you recommend getting a letter from the biggest name possible, or would a small community program director be as good if they wrote a good letter?

  • Clinical letters are more important - where you can show your strengths more easily than in radiology, i.e., by shadowing and having a solid knowledge and participation in conferences in radiology. That said a a letter from a research mentor is important. If going into IR, should get an IR letter and DR letter - if one is also a research mentor, it makes it easier. So 1-2 clinical, 1-2 DR/IR, 0-1 research (whether radiology or not)
  • Good personal letter more important big name who doesn't know you
Do you recommend away rotations? particularly if your home program is a small, community one?
  • No, not unless you need to go to a certain locale. Can consider if you are applying to IR. It is hard to impress on a radiology rotation as you don't really have patient responsibilities.

What are some things you see residents coming to learn toward the end of residency they wish they had known when they first started out residency training?

Wow, that's a loaded question... but there are a few things that come to mind...

Slow and steady wins the race - read a little every day. You won't see what you don't know.​
  • Read a med student radiology book cover-to-cover (e.g., Learning Radiology) just before your first year, or during your first month to get a good skeleton of knowledge.
  • Read a smallish book (~300 pp) for every rotation during your 1st year so you add some flesh to skeleton of knowledge by the end of the year. Once you have a skeleton, random facts you pick in conference or day-to-day cases stick better.
  • The next years, read larger books and review articles (e.g., Radiographics) to add more flesh and cement the knowledge
See as many cases as possible - pick up cases proactively - better to go fast and see more cases than go slowly over each case - the law of diminishing returns applies as you will probably not pick up more findings if you spend more time.​

Does independent call during residency really make for better residents and overall better radiologist? What have you found to be the ideal call set up?

This is a controversy throughout all residencies - 24/7 coverage by IM and peds hospitalists, EM etc.​
  • If you are alone, you have to make decisions on your own, will become more confident. You will look things up and work faster.
  • However, you will make mistakes - and even if it makes a difference to only one patient a month, it is one patient a month.
Probably the best way - be as independent as your level will allow you, picking up as many cases as possible, and of course asking for help as needed. The attending can be out of the room or out of the way - but will help as needed.​

Job market is currently improving, but still has a way to go. How does the job market affect the dynamic of the residency program, if at all?

Didn't notice any difference in the dynamics.​

What do you feel are the top 5 traits that make for a successful radiology resident?

Probably similar for all physicians - and likely everyone will have their own lists.​
  • Professionalism
  • Empathy and kindness - you often see only the images of patient suffering, so even more important
  • Knowledge - Lifelong learning - lots to read in radiology (similar to pathology)
  • Work ethic and going the extra step
  • Curiosity & Passion
I think I answered all the questions - if I missed something, or you need clarification, let me know.
 
Last edited:
  • Like
Reactions: 10 users
Members don't see this ad :)
"6. Our residency essentially filters out applicants who are IMGs—to be honest, I think there are some outstanding candidates in this group particularly from those individuals who are not from the US, but the problem is that it is really difficult to find those for me because we don’t use test scores as much in our ranking process. Communication skills are very important to us, and that can be a sticking point for some IMGs who did not grow up in the US that we don’t understand until the interview, and I don’t want to waste people’s time with interviews if there is a low chance of success. On the other hand, we understand there are some life circumstances and other legitimate reasons why some applicants who are US based ended up doing medical school internationally. So I download these into my spreadsheet, dig deeper at maybe the candidates with board scores above 250 to see if I recognize the school, review publications, special experiences, special circumstances, etc. Sometimes a colleague will ask me to look carefully at a person that someone in their field has highlighted for them. We do interview a few IMGs every year (< 5), and some are quite good. However, they face hurdles all along the way in our ranking process—just being honest.

7. We do the same as #6 for DO candidates. We do interview a few every year (< 3), but we believe there is a penalty for our program in future applicants if we have a number of DO residents, mainly because there is the perception that we couldn’t attract the best MD candidates. It’s unfortunate for some DO students who are going to be great, but it is reality."



Could you please go into greater detail on this process and why it is acceptable to eliminate DOs now that the residency programs are merged. I do not want to get into an MD vs. DO discussion at all. I'm just wondering how this thought process of eliminating well-qualified DOs on the basis of their degree can continue to go forward in the new GME environment. I would also like to hear more about the "penalty" your program receives if you have DO residents.
 
  • Like
Reactions: 1 user
"6. Our residency essentially filters out applicants who are IMGs—to be honest, I think there are some outstanding candidates in this group particularly from those individuals who are not from the US, but the problem is that it is really difficult to find those for me because we don’t use test scores as much in our ranking process. Communication skills are very important to us, and that can be a sticking point for some IMGs who did not grow up in the US that we don’t understand until the interview, and I don’t want to waste people’s time with interviews if there is a low chance of success. On the other hand, we understand there are some life circumstances and other legitimate reasons why some applicants who are US based ended up doing medical school internationally. So I download these into my spreadsheet, dig deeper at maybe the candidates with board scores above 250 to see if I recognize the school, review publications, special experiences, special circumstances, etc. Sometimes a colleague will ask me to look carefully at a person that someone in their field has highlighted for them. We do interview a few IMGs every year (< 5), and some are quite good. However, they face hurdles all along the way in our ranking process—just being honest.

7. We do the same as #6 for DO candidates. We do interview a few every year (< 3), but we believe there is a penalty for our program in future applicants if we have a number of DO residents, mainly because there is the perception that we couldn’t attract the best MD candidates. It’s unfortunate for some DO students who are going to be great, but it is reality."



Could you please go into greater detail on this process and why it is acceptable to eliminate DOs now that the residency programs are merged. I do not want to get into an MD vs. DO discussion at all. I'm just wondering how this thought process of eliminating well-qualified DOs on the basis of their degree can continue to go forward in the new GME environment. I would also like to hear more about the "penalty" your program receives if you have DO residents.

To be honest, I think the PD is basically saying what we all know to be true anyways. Namely, that programs that take IMG's/DO's are perceived to be less competitive than others. Therefore, they become less attractive to stronger AMG candidates. This is why programs would rather go unfilled (as many have recently) than to suffer a ding to their reputations.

The penalty is perception, which directly affects reputation. This is the bitter truth, but it is reality. This is why we will literally leave no stone unturned into getting into an American med school.

It's nice to hear someone finally acknowledge this point.
 
Last edited:
  • Like
Reactions: 7 users
"6. Our residency essentially filters out applicants who are IMGs—to be honest, I think there are some outstanding candidates in this group particularly from those individuals who are not from the US, but the problem is that it is really difficult to find those for me because we don’t use test scores as much in our ranking process. Communication skills are very important to us, and that can be a sticking point for some IMGs who did not grow up in the US that we don’t understand until the interview, and I don’t want to waste people’s time with interviews if there is a low chance of success. On the other hand, we understand there are some life circumstances and other legitimate reasons why some applicants who are US based ended up doing medical school internationally. So I download these into my spreadsheet, dig deeper at maybe the candidates with board scores above 250 to see if I recognize the school, review publications, special experiences, special circumstances, etc. Sometimes a colleague will ask me to look carefully at a person that someone in their field has highlighted for them. We do interview a few IMGs every year (< 5), and some are quite good. However, they face hurdles all along the way in our ranking process—just being honest.

7. We do the same as #6 for DO candidates. We do interview a few every year (< 3), but we believe there is a penalty for our program in future applicants if we have a number of DO residents, mainly because there is the perception that we couldn’t attract the best MD candidates. It’s unfortunate for some DO students who are going to be great, but it is reality."



Could you please go into greater detail on this process and why it is acceptable to eliminate DOs now that the residency programs are merged. I do not want to get into an MD vs. DO discussion at all. I'm just wondering how this thought process of eliminating well-qualified DOs on the basis of their degree can continue to go forward in the new GME environment. I would also like to hear more about the "penalty" your program receives if you have DO residents.

The way I interpreted it was that the penalty was the perception of a weaker program that "had to use DOs to fill."

Edit: Phone didn't show me above post that says the same.
 
  • Like
Reactions: 2 users
This thread is great.
I was an interviewer for my residency this past cycle, and I can confirm that there's just too much stuff in each and every single application to go in-depth into research and experiences. My impression has been that applicants obsess over these aspects of the applications far more than the program spends evaluating -- not to say that they're not important, but they definitely take a backseat to quite a few other things.

On the flip side, I spent quite a bit more time on the LORs, and phrases such as "top 5% of all students I've worked with" really stand out. Everyone seems to think that he/she has excellent LORs, but let's face it...most letters appear excellent and filled with superlatives at face value, and there are particular phrases that say much more than paragraphs of fluff.
 
Last edited:
  • Like
Reactions: 2 users
So I just want to put this out there: the PDs are going to be honest and forthcoming in answering our questions. This means what they say may hurt some
peoples feelings, and we may want to react (myself included).

We have a huge asset in the PDs.

Lets please be good to them and just allow them the room to breathe and answer our questions. We dont want to push them away.

We dont know them, and they dont know us. Lets keep our own feelings aside as best as we can, lest we lose this wonderful privilege.
 
Last edited:
  • Like
Reactions: 10 users
Greetings and thanks so much for doing this.

My one and only question to the both of you is this: How do you view the "number 1" email from applicants. Have these emails caused you to move applicants up your rank lists? Especially if you knew these emails were genuine (email backed up with a phone call)?
 
Knocking out some of the easier questions tonight before I settle in to some other stuff--I will definitely try to answer all questions (eventually), its just that my style tends to be as thorough as possible in my initial response to some of the really big questions.

My one and only question to the both of you is this: How do you view the "number 1" email from applicants. Have these emails caused you to move applicants up your rank lists? Especially if you knew these emails were genuine (email backed up with a phone call)?

Love these #1 notes. It doesn't change our rankings at all--very honest about this--but it's always nice to get love letters. Having said that, I explicitly tell applicants they don't have to send me any love notes after the fact. On the other hand, I really don't like the "loved you, will rank you highly" notes--I know that we aren't that person's #1. It also doesn't change our rankings, but I want everyone to rank us #1--so it bums me out a little.

On the flip side, I spent quite a bit more time on the LORs, and phrases such as "top 5% of all students I've worked with" really stand out. Everyone seems to think that he/she has excellent LORs, but let's face it...most letters appear excellent and filled with superlatives at face value, and there are particular phrases that say much more than paragraphs of fluff.

Some LORs are indeed helpful--most are not. The really good ones do help applicants in the "33%" in our system who aren't no-brainers for interview based on their "non-interview" score (again, this is just our system).

Could you please go into greater detail on this process and why it is acceptable to eliminate DOs now that the residency programs are merged. I do not want to get into an MD vs. DO discussion at all. I'm just wondering how this thought process of eliminating well-qualified DOs on the basis of their degree can continue to go forward in the new GME environment. I would also like to hear more about the "penalty" your program receives if you have DO residents.

Trying to be as honest as possible. Here are the problems for our program with respect to most DO applicants and certainly all IMG applicants:
1. The perception (again, perception) that we struggled to fill the program if a number of our residents are DO/IMG. Without trying to give away too much, I'm not at Mallinckrodt--my program isn't one of those 15+ residents/year beasts that can have an IMG or DO and everyone decides "dang, that person must be awesome"--instead, for us, because we have a smaller program, the impression will be, "why couldn't they fill with AMGs". Realize that this is not just a perception among applicants--I'd have explain to all my faculty (none of whom are DOs) that we really wanted this DO--they are going to think we had a bad Match.
2. More importantly (seriously)--the DO schools do a terrible job stratifying their medical students. This is the real problem I have with understanding how to asses my DO applicants. Even worse for the IMGs. More than 50% of a typical DO class (sometimes 80%, it seems to me) get "straight A's" in the clinical years. The written commentaries on performance are woefully short (one sentence, maybe two). I don't know the radiologists that have worked with them, so I don't get any familiar letters that would boost an applicant. Many of the radiology departments that the DOs have worked in aren't particularly academic, so they haven't impressed in research. Since I really don't care if you get a 240 or a 270 on your USMLE, it becomes that much harder for me to be impressed by a DO student who got 270 on their boards (by the way, I do care if you get less than 240, so please do as well as possible on your USMLE). Even though we purposefully "overinterview", I have far more applicants than individuals I can invite. If you are a DO, your best bet to get into my residency would be to have an advocate that I'm familiar with call me and tell me you walk on water. I'd happily go to bat for you then, with my faculty and with future applicants.

With regard to some other easy questions:
1. Choice of internship and location has no bearing on how I'm going to rank you. It definitely might have an impact on whether it helps you achieve your goals (more on that later).
2. We don't care if you are also looking at IR/DR combined programs. We currently don't have one (but will), so I wouldn't really know if you were applying to IR programs also, but it becomes obvious from the personal statement and during the interview.

More later.
 
  • Like
Reactions: 13 users
What should applicants be looking for in a residency?

I believe the most important question an applicant should consider is whether or not he/she feels the program can inspire them to be the best version of what that person wants for himself/herself and also help him/her achieve those goals, while at the same time keeping him/her emotionally happy and mentally engaged. This is a highly personal calculation and requires some serious introspection.

Let’s say one applicant has always wanted to be a neuroradiologist, is just inherently more interested in diseases that affect the brain/spine, and knows that he/she is unlikely to waver from that goal. The person doesn’t really know or may not care if he/she is practicing in an academic center or in a high functioning private practice, but he/she wants to focus exclusively on being the best clinical neuroradiologist he/she can be. The best program for that applicant is one that has neuroradiologists that are inspiring, that want to mentor new radiologists, that can help that person get the best neuroradiology fellowship in the geographic area that the applicant is most interested. Do these neuroradiologists do interesting research?—great—that’s going to be important for the applicant when they compete for fellowships! Do they give CME talks to other radiologists?—great! Do the neuroradiologists have connections at the national level, which lends them clout in recommendations? Are they receptive to mentoring?—all the better. Do the residents at that institution like the neurorads?—critical. Does the institution have an interest in being recognized for excellence in neurosurgery and/or neurology?—awesome.

Another applicant is picking radiology for lots of reasons, but foremost is his/her desire to have one-on-one teaching interactions with clinicians/fellows/residents. Maybe this person recognizes that he/she is particularly skilled at tutoring, or explaining concepts to others. He/she likes the idea of becoming an “instructor” in an area that he/she has developed expertise. Doing that on a daily basis in combination with "cranking out the cases" would make it all interesting. Does the program have teachers who excel in this, that can serve as role models? Does the resident get a chance to form real connections with faculty, or is the program so big as to make this difficult? Are there resources (illustrators, research electives) that allow the individual to create a teaching portfolio? Is expertise valued at this program at the attending level? Is there anything about the program that can inspire this individual to keep moving forward with this notion of wanting to instruct? Or does everyone in the program eventually just go into private practice because there are great jobs in the area which is so highly seductive and desirable to others?

Another applicant wants to be a private practice radiologist functioning at a high level in the Rocky Mountains. He/she wants to spend their 4th year doing concentrated electives in “something” (they aren’t sure or don’t really care as to exactly what yet) but the goal is to secure a private practice job in Colorado, almost certainly after a fellowship in whatever is hot at the time. Is the program geographically close to the target area? Does the program have a name brand that will open doors for the individual in terms of securing whatever fellowship might advance that goal (even if the person isn’t sure of the field)? Is the program set up to give you more independent call in the 4th year than the 2nd year, which means the person will function more like a private practice radiologist as they near completion of training when they are most skilled rather than at the beginning of training when they are just learning the ropes (why do I think this important?—another topic, another day).

There might be the applicants who are open to anything in the imaging world. They aren’t absolutely sure of where they want to end up geographically, or what fellowship they want to do. They just have come to understand that they think they like the role of the radiologist in the US health system—a consultant, selecting and interpreting appropriate imaging, providing interventions where they are able, quickly dismissing the normal cases and perseverating and “collecting” the interesting cases like a kid collects sports cards (“wait til I show you my Ernie Banks card…”). Does the program see interesting cases? Is there a culture of “show and tell”? Do clinicians at the institution value the radiologists? Are there opportunities to present cases at multidisciplinary conferences? Are the residents proud of their department?

Consider applicants who are socially gregarious and most passionate about being in a team. They enjoy being the life of the party. Sharp as a tack, they are going to do what they need to do to be good at radiology, but they’d feel best if they developed a sense of camaraderie among their fellow residents. It’s fun to come in every day and schmooz with the pals! Is the residency program large enough to satisfy this person’s social inclinations. Are there “intramural” sports teams among the residents? Do the residents have an active social life together? Does the program have an alumni brand that will satisfy this person?

So—do some introspection to understand where your passion lies, and look for values/structure/resources in programs that will keep that flame alive, and hopefully nurture it! Be honest with yourself about the things that matter most. At this point in the game, aim high—you might not hit all your goals, and your goals might change, but rank programs in a way that gives you the best chance at achieving YOUR personal goals, not some generic version of what the goals should be for a radiologist.

Beyond an alignment of the program’s strengths with your particular passions, look for program organization and thoughtfulness. Can they explain why they do what they do? Some programs have a plan/method. Examples: Do you do the same rotations as others in your year, or is it haphazard as to whether you get “county US” year 1 and someone else gets “county US” year 3. Why do they organize call the way they do—is it just about coverage, or is there a rationale why you are taking a certain type of call R2 year vs. R3 year vs. R4 year.

I do think it is important to consider the “non-radiology” aspects of life. Commuting, cost of living, spouse/significant other preferences, climate, etc. all can have a huge impact on your happiness, which impacts your ability to become that “best version of you”. For some people, these factors will be more important that many “radiology” factors.

Notice I didn’t mention some things that most medical students think are differentiators in terms of programs, such as:

1. Moonlighting opportunities. Understandably desirable to some extent, but this shouldn’t be a primary reason to select a program—IMO.

2. Volume in and of itself. I agree wholeheartedly with an earlier comment by Radiology_Advisor—“See as many cases as possible”. However, I would add the proviso that you should “see as many cases as possible developing a process that allows you to eventually miss as little as possible”. What I mean by that is that call experiences in which residents learn to quickly deliver prelim reports and do this over and over every night on call makes you very good at prelim reporting—but not so good at doing what radiologists do, which is reporting the case with all necessary nuances and considerations about all findings, including incidental findings. If you “see as many cases as possible” but the person who checks your cases is overwhelmed with work, or not specialized in the area of the examination, you may not learn a thing. At our institution, I’ve seen fellows who were residents at some programs that were “volume heavy” but with a process emphasizing prelim reports come to us and get completely tongue-tied and bogged down after the first few weeks when taking “attending” call at our institution, when their reports are final and all the mistakes they make get pointed out to them by way of our subspecialty review process—“no DVT, but missed the fact that the waveform shows evidence of a central venous stenosis which accounts for the leg swelling, and for which a venogram with possible angioplasty would be in order—oops”. Don’t get me wrong—the best residents are those that can plow through the volume but still have developed that “sense” of knowing when and how to dig into the EMR more to explain a particular finding or a discordant clinical symptom, those that will diligently look for relevant comparison exams (quickly), those that will get off their butt and check to see if the patient really has or does not have a sonographic Murphy sign since that will be better than just slapping on the “clinical correlation recommended” phrase—and if the program is so “volume heavy” that it doesn’t allow you to develop these skills progressively as you get more advanced in training, then the volume might be counter-productive. It reminds me of the old adage: “miss ‘em fast”. Not a motto one should strive to adopt in training.

3. Interview dinner experience. Really?
 
Last edited:
  • Like
Reactions: 18 users
This thread is incredibly helpful, really wish I had this when I was applying!


Sent from my iPhone using SDN mobile
 
This should be sticky thread!
 
  • Like
Reactions: 1 user
To the PDs: It appears that Step scores (Step 1 in particular) help offer some kind of prognosis on the probability of passing the boards. My question, is there any consistent relationship between USMLE scores and resident performance in radiology?
 
I have a little bit of a different perspective about residency selection, as a current applicant to IR.

I realized the most important aspect of medical training is to leave the MOST amount of doors open for the next stage of training. In general, the larger, more well known program you go to, the more doors remain open.

While many radiology PDs will tell you that "we always match people to Harvard or UCSF", that is both true and false in a sense. It's true because every year people will be able to get to UCSF or MGH from ALMOST any program.

What has left unsaid is that choices of residency really begin to matter when the competitiveness of a specialty begin to increase. It's simple, really, all being equal between two applicants, the one who went to a well known program will have more boxes checked.

I went to a small community radiology residency program, busted my asses during residency and did a lot of research as well as having excellent letters and good (240+) step 1 and 2. Meanwhile, a coresident came from a DO school and did no research due to having a lot of family responsibilities

He is applying to neuroradiology right now, and is deciding between big names like Harvard or Yale. Meanwhile, I am appying to IR and I did not get many big name invites. While I am incredibly happy with the fellowship programs I am getting and hopefully will match, once a subspecialty is competitive, it absolutely matters to where you go for residency because your letters will come from big names in the field PDs know and trust.

Plus, you never know where life will take you. You may decide you suddenly want to do IR. You may meet a woman who is hellbent to go back to live in NYC where she grew up.

So if I was back in my shoes during med school days, I would strongly consider going to where the program has the strongest name. Many programs provide very strong training but those with good names are a lot fewer.

Name will carry you to places and leave the most amount of door open.
 
  • Like
Reactions: 2 users
I have a little bit of a different perspective about residency selection, as a current applicant to IR.

I realized the most important aspect of medical training is to leave the MOST amount of doors open for the next stage of training. In general, the larger, more well known program you go to, the more doors remain open.

While many radiology PDs will tell you that "we always match people to Harvard or UCSF", that is both true and false in a sense. It's true because every year people will be able to get to UCSF or MGH from ALMOST any program.

What has left unsaid is that choices of residency really begin to matter when the competitiveness of a specialty begin to increase. It's simple, really, all being equal between two applicants, the one who went to a well known program will have more boxes checked.

I went to a small community radiology residency program, busted my asses during residency and did a lot of research as well as having excellent letters and good (240+) step 1 and 2. Meanwhile, a coresident came from a DO school and did no research due to having a lot of family responsibilities

He is applying to neuroradiology right now, and is deciding between big names like Harvard or Yale. Meanwhile, I am appying to IR and I did not get many big name invites. While I am incredibly happy with the fellowship programs I am getting and hopefully will match, once a subspecialty is competitive, it absolutely matters to where you go for residency because your letters will come from big names in the field PDs know and trust.

Plus, you never know where life will take you. You may decide you suddenly want to do IR. You may meet a woman who is hellbent to go back to live in NYC where she grew up.

So if I was back in my shoes during med school days, I would strongly consider going to where the program has the strongest name. Many programs provide very strong training but those with good names are a lot fewer.

Name will carry you to places and leave the most amount of door open.

I think most of us would agree with you on many points.

But please keep in mind this is an AMA thread.
 
  • Like
Reactions: 1 user
This thread is pretty amazing

1.) What do you think is the most common mistake first year residents make?
2.) Has an interview solely made or broken an applicant? If so, how?
3.) Does the input from residents on the dinner the day before come into play?
 
Hey everyone, just FYI, I'm not likely to have the time to engage in a lot of private conversations. I apologize in advance if you PM me and I don't respond. Having said that, with permission of the poster, I am copying a recent conversation here so it can be answered for all.

"I find it unfair that great U.S. IMG applicants can be screened out simply because of where they went to school. Would you mind explaining why a U.S. IMG who is stronger on paper can be discarded over a U.S. student just because of where they went to school?"

You are correct that if I had a different set of circumstances, I might make the investment to find those best US IMG students. You are correct that our program gets a lot of applicants based on reputation and other factors. We get more than 100 applicants for each spot. That's just too many for me to dive into. So what happens is that I use the USMLE filter I described--but I'm still left with far too many applications than I can review. I honestly do not believe that you will necessarily be a better radiologist if you scored 260 compared to 240 on USMLE. (Parenthetically, I do think radiology is one of those fields in which you need to be able to retain a lot of facts to get your work done quickly--yes, everything can be looked up, but you don't have time in practice, so it helps to have a mind that can retain facts--so doing better on USMLE is good, but doesn't need to be 260). Anyway, since in my mind a candidate with 240 is potentially just as good as a candidate with 260, then I take the time to dive into the 3rd year clerkships and quick review of the Dean's letter--but I don't have time to do that for everyone. Because I have a plethora of candidates with USMLE scores at/over 240, and because I interview "local" candidates and candidates brought to my attention by my colleagues with USMLE scores as low as 220, I just can't look at all the IMG candidates with high USMLE. I reviewed my spreadsheet for this current cycle--I had 20 IMG candidates with USMLE at/over 250. My notes show that I did dive into about 10 of the applications whose Step 2 scores were over 260--we ended up interviewing 3 of those candidates this last cycle. Their interviews were good, but each candidate had an uphill battle during our ranking discussions (which I have not yet described)--we have ranked all 3 candidates, but based on our historical match level, we will very likely fill prior to their number.

Most of what you said above makes sense, I suppose I am just biased. Would you mind elaborating more on the following (feel free to answer this in the main thread):

"Their interviews were good, but each candidate had an uphill battle during our ranking discussions (which I have not yet described)."

For your program, it seems that all efforts made by an IMG would be futile unless a respected radiologist that you are familiar with gives you a call (even then, seems unlikely). But what suggestions do you have for U.S. IMGs in terms of standing out amongst their peers when applying to other top-tier programs? What qualities made you give the 3 IMGs you granted interviews to a chance?


As I have yet to describe, our program gives more weight to factors we try to glean from the interview. As you might imagine, not all of our 7 selection committee members see these factors exactly the same way when evaluating any one candidate. The "uphill battle" is that every one of the selection committee members balances their interview assessment against the "perception" card, and I know from seeing this play out that the IMG candidate's stock goes down, especially given the recent past in which more radiology programs have had to turn to the SOAP--my committee is justifiably worried about perception. Like it or not, it is a real issue. Since I see this all the time, why waste my committee's time presenting them with candidates that they are going to ding due to perception? As a result, I am increasingly hesitant to bring most of the IMGs in for an interview, because I don't want to waste their money or time. The 3 candidates chosen were all US IMGs with super high step 2 scores, great clerkship performance, with great letters. They also clearly explained in their application how and why they ended up going abroad for school in what came across as a sincere life circumstance issue. I think they are going to do fine in the Match--as others we've interviewed in the past have done--but it's just tough at my program. Again, being honest.
 
  • Like
Reactions: 8 users
Oh, and one more thing--all 3 candidates had a tie to our region. In the past, we've had a couple of IMGs who would indeed have matched with us based on how we ranked them--in fact, one year we ranked one first, that person was a star--but they went to pretty good east coast programs.
 
PLEASE please please do not abuse the kindness of our volunteer PDs and PM them, especially things that can be answered here.

It's not fair to them, can be overwhelming and is also unfair to others who might have the same question (which could be answered here).


Sent from my iPhone using SDN mobile
 
  • Like
Reactions: 9 users
Okay, we got a good glimpse into your pre-interview selection process. Great, we made it to the interview. Now what? What are you looking for on interview day? Does anything I say really matter as long as I'm a well-adjusted non-creep?

Edit: and when you send the holy grail "you are ranked to match" email, what exactly does that mean?

Edit 2: honestly curious because it's a predicament I'm personally in and would appreciate some PD weigh-in. I doubt I'm the only person dissecting post-interview communication letter by letter. No disrespect intended.

Sent from my iPhone using SDN mobile
 
Last edited:
What are some pitfalls for applicants to avoid, in both the application and the interview?

You mentioned that "red flags" can be a deal breaker, do you have any examples that would preclude an applicant from an interview (given that they met your other criteria for interview - scores, AOA, regional, honors on clerkships, etc.)?

Once applicants make it to the interview stage, what are some qualities or factors that determine rank order? Anecdotes may be helpful here. How does an applicant end up at the bottom of the list vs. the top?


Sent from my iPhone using SDN mobile
 
Okay, we got a good glimpse into your pre-interview selection process. Great, we made it to the interview. Now what? What are you looking for on interview day? Does anything I say really matter as long as I'm a well-adjusted non-creep?

Edit: and when you send the holy grail "you are ranked to match" email, what exactly does that mean?

Edit 2: honestly curious because it's a predicament I'm personally in and would appreciate some PD weigh-in. I doubt I'm the only person dissecting post-interview communication letter by letter. No disrespect intended.

Sent from my iPhone using SDN mobile

What is the point of this? Algorithm's already been run and our fates are already sealed manbrodude...
 
Don’t get me wrong—the best residents are those that can plow through the volume but still have developed that “sense” of knowing when and how to dig into the EMR more to explain a particular finding or a discordant clinical symptom, those that will diligently look for relevant comparison exams (quickly), those that will get off their butt and check to see if the patient really has or does not have a sonographic Murphy sign since that will be better than just slapping on the “clinical correlation recommended” phrase—and if the program is so “volume heavy” that it doesn’t allow you to develop these skills progressively as you get more advanced in training, then the volume might be counter-productive. It reminds me of the old adage: “miss ‘em fast”. Not a motto one should strive to adopt in training.

Completely agree - RadiologyPD said it much better than I did. I don't think that this can be emphasized enough:
A radiologist is a consultant - and not only should find the findings, but should put the entire story together and come up with a diagnosis or differential diagnoses, and solid recommendations.
 
  • Like
Reactions: 1 users
These are great questions. I'll try to tackle them in depth, but it's going to take a while for me.

I thought it might help to understand process for selection for interview at my place. You can glean what I'm looking for at this stage of the application process--it only applies to selection for interview, not ranking after interview.

Here's what I do:

1. I download the ERAS application data into an excel spreadsheet which allows me to create custom parameters and is easier for me to filter/sort and quickly review. ERAS allows you to download certain parameters. Each applicant is a row, and I type notes and create formulas into custom columns that I create.

2. I personally select the resident applicants that we will interview from the hundreds of applications we get. It’s just too slow and hard to do this with a committee. At my old institution, this was also done by essentially 1 person (not me back then), but I don’t know if they have changed that. I have a “2nd reviewer” for borderline cases, maybe I use that for 3-4 applications each year.

3. If an applicant has taken USMLE Step 2, I average that score with Step 1. If an applicant hasn’t’ taken USMLE Step 2, I add 5 points to their Step 1 score and create a “derived Step 2” score. In our applicant pool, the average applicant has a Step 2 score that is about 7-10 points higher than Step 1. So it hurts you a little if you haven’t taken Step 2 because my assumed score for you is not as high as it statistically would be if you are the average applicant.

4. Turns out I apply a formula to the “combined USMLE score” that discounts super-high achievement. Essentially, as your score on Step 1 or Step 2 gets higher from 250, you get less added points. For example, someone who gets 250 on Step 1 and Step 2 has a combined score of 500 in my system. However, someone who gets 265 on Step 1 and Step 2 has a combined score of 515 in my system, not 530. I don’t want super-high achievement on USMLE to dominate an applicant’s eventual “overall score”, or to make up for lower clinical grades and interview scores. This last year, in our program, our applicants had a mean combined USMLE score of 494, and a median of 498, with a standard deviation of 19. Remember, those scores have been adjusted to discount performance substantially above 250 in a formulaic way that progressively discounts numbers the farther above 250.

5. We filter out the applications based on USMLE scores, but we use a really low threshold—in our case, we use a “soft” 220 USMLE step 1 score, which generally means I will consider applicants in the 215-220 range based on strength of school and how well they did on step 2, as well as other factors. Right now the average Step 1 score for all medical students is about 228 or so, I believe, so allowing someone to be as low as 215 in our case means you don’t necessarily have to be book smart to get into our program. However, radiology boards is now a computerized test, and we don’t want to worry about residents who may become outstanding radiologists but who might struggle with tests. There is too much of a penalty for our program in future applicant perception if one of our residents fails the boards.

6. Our residency essentially filters out applicants who are IMGs—to be honest, I think there are some outstanding candidates in this group particularly from those individuals who are not from the US, but the problem is that it is really difficult to find those for me because we don’t use test scores as much in our ranking process. Communication skills are very important to us, and that can be a sticking point for some IMGs who did not grow up in the US that we don’t understand until the interview, and I don’t want to waste people’s time with interviews if there is a low chance of success. On the other hand, we understand there are some life circumstances and other legitimate reasons why some applicants who are US based ended up doing medical school internationally. So I download these into my spreadsheet, dig deeper at maybe the candidates with board scores above 250 to see if I recognize the school, review publications, special experiences, special circumstances, etc. Sometimes a colleague will ask me to look carefully at a person that someone in their field has highlighted for them. We do interview a few IMGs every year (< 5), and some are quite good. However, they face hurdles all along the way in our ranking process—just being honest.

7. We do the same as #6 for DO candidates. We do interview a few every year (< 3), but we believe there is a penalty for our program in future applicants if we have a number of DO residents, mainly because there is the perception that we couldn’t attract the best MD candidates. It’s unfortunate for some DO students who are going to be great, but it is reality.

8. One of the filters I use to select who to interview is 3rd year core clerkship performance. This is a really tough metric, since the schools are all over the place. Believe it or not, I spend the time to create a “translation formula” that looks at the % of Honors/High Pass/Pass from each US medical school from which we receive an application, so I can compare the performance of students from different schools—even then, it’s not easy and likely not accurate. For example, just looking at my spreadsheet for this year, I see that for the core clerkships that we review, at the University of Central Florida 52% of students got Honors and 48% got High Pass (no one just passed), whereas at Florida International University 14% got Honors and 29% got High Pass. I have a convoluted formula that tries to “normalize” this data, so that the student at FIU that got High Pass is given the same number of points as the student at UCF that got Honors.

9. Since I have to actually open up the application to get the 3rd year clerkship performance and % honors/high pass/pass data, I do quickly jot down a few notes about the candidate at this time—I’ll jot down a few sentences about the candidate about their particular timeline, skimming the personal statement, looking quickly at the research history, etc. I quickly look at the Dean’s letter (if available) to look for red flags (repeat courses). I do put down what “quartile” the Dean’s letter says you are in. I don’t have time to review LORs at this point UNLESS I can tell that the candidate is probably going to be on my “borderline” for selecting for interview. For example, here is a typical fictional set of notes that I might jot down in my “comment” box for a candidate at this point: “Brown undergrad; 1 yr gap spent as research intern for startup and also doing volunteer work; PS specifically mentions us”. In the “Dean’s letter” box, I might put “2nd quartile”.

10. My spreadsheet combines the “3rd year clerkship score” with the USMLE average score (either real or derived) in a way that weights the clerkship score. This gives each applicant a “non-interview” score—that is, their score without consideration of the interview. As you will see later, the eventual evaluation of a candidate relies more on the interview than this score. But this is the metric that helps us decide who to interview.

11. I sort my spreadsheet using the “non-interview” score to decide who to interview. For about 67% of our interview slots, I just take them from the top. For the final 33%, I use a different lower threshold for interviewing applicants who are considered “local” (from our med school and schools within about 100 miles of our urban program), “regional” (about 500 miles), and “national” (everyone else). The reason we do this is because we find that applicants are more likely to not cancel interviews and match with us if they are local or regional. We also don’t want to piss off our school/the local schools and not interview their students—to a degree. We won’t interview if someone is really not going to be up to snuff based on performance. If a national candidate has ties to our area that are obvious in the application (I look at permanent address and undergraduate location), then they get put in the local pile. Similarly, if the applicant did a rotation with us, we consider them in the local pile even if they aren’t. However, we don’t take that many medical students outside our own medical school for rotations. Along with the varying thresholds based on geography, I look at my comments and the “Dean’s letter comments” to decide who to select for these final 33%.

12. If the applicant is AOA, they almost always get an interview. Turns out they always score above my threshold on the non-interview score anyway (makes sense, since AOA status is generally a function of traits that are well reflected in the USMLE scores and core clerkship grades). However, we sometimes have an AOA student who I end up not interviewing, because of something in the application that is a red flag that I can easily see from my spreadsheet (repeating a course, something in the Dean’s letter).

13. We typically “overinterview” in our program—basically, interview about 15-20 applicants for every spot, even though we typically fill our spots at the 7-10 applicant/spot filled mark—and even then, about 3/4ths of our class is filled at the 5 applicant/spot mark. The reason we do this is because we don’t trust that our combined “USMLE + clinical clerkship” score is so precise that we can rely on it, and we sometimes find that applicants we end up ranking fairly high would not have been offered an interview with us if we had not “overinterviewed”.

So, a few things should be evident so far, regarding “how to get an interview” at my program:

1. Do well on USMLE tests, but no need to ace it. Does it help to get a 270 vs. a 250?—not really.

2. Do well on 3rd year core clerkships.

3. Be local or communicate your connection to my community in some way—it lowers the threshold for you getting an interview. Say it in the personal statement if it is true. Even then, you might want to email me in advance if that local connection isn’t obvious in the application.

4. I don’t have time to consider whether you decided not to do a rotation with us at the “select for interview” stage. I don’t have time or an easy way to consider the strength of your research record. I don’t have time to look at your Dean’s letter in depth beyond just trying to make sure there is no coded “red flag” and to understand your relative performance. I don’t have time to consider your extracurricular activities.

Once you get selected for interview, the selection metrics become more nuanced—another long discussion for another day.

Can you tell us the Honors/High Pass/Pass ratio for University of South Florida Morsani College of Medicine in Tampa and University of Florida in Gainesville?
 
Wasn't considering radiology but this thread may have put it on the map for me. Mods really outdid themselves with this one; really takes away most of the guess work and hearsay. @Lee Is there any way, any way at all, to get PDs for the other specialities to do the same? There is something to be said for the level of transparency here. @RadiologyPD and @Radiology_Advisor thank you for taking the time. I do think you may want to remain anonymous throughout though unless you want to get customized applications tailored to highlight everything in this little cheatsheet you have so graciously offered us.
 
Last edited by a moderator:
Can you tell us the Honors/High Pass/Pass ratio for University of South Florida Morsani College of Medicine in Tampa and University of Florida in Gainesville?
What? I don't think this is the appropriate place to be asking these questions, lol.
 
  • Like
Reactions: 9 users
Okay, we got a good glimpse into your pre-interview selection process. Great, we made it to the interview. Now what? What are you looking for on interview day? Does anything I say really matter as long as I'm a well-adjusted non-creep?

Edit: and when you send the holy grail "you are ranked to match" email, what exactly does that mean?

Edit 2: honestly curious because it's a predicament I'm personally in and would appreciate some PD weigh-in. I doubt I'm the only person dissecting post-interview communication letter by letter. No disrespect intended.

Sent from my iPhone using SDN mobile
Try not to worry. I wouldn't let yourself be in a predicament. Lists are turned in and there is literally nothing you can do.
 
So getting back on track....
you mentioned there isn't enough time to really dive into the research experiences for most applicants, so other than the personal benefit applicants get from the research, how does that bolster their application beyond just a +1 to presentations/abstracts/publications?
I guess a better way of asking this is: what stands out to you while quickly looking at the research history?

As someone who is not from an academic center I've had to scrap for opportunities to do research in radiology, so although I know it's just a small portion of the app it's just something I worry about being from a newer med school.

Thanks so much @RadiologyPD and @Radiology_Advisor for the best thread I've seen in a long time.


Sent from my iPhone using SDN mobile
 
So getting back on track....
you mentioned there isn't enough time to really dive into the research experiences for most applicants, so other than the personal benefit applicants get from the research, how does that bolster their application beyond just a +1 to presentations/abstracts/publications?
I guess a better way of asking this is: what stands out to you while quickly looking at the research history?

As someone who is not from an academic center I've had to scrap for opportunities to do research in radiology, so although I know it's just a small portion of the app it's just something I worry about being from a newer med school.

Thanks so much @RadiologyPD and @Radiology_Advisor for the best thread I've seen in a long time.


Sent from my iPhone using SDN mobile
My impression was that this was only in the pre-interview phase. My guess is research may become much more important when ranking applicants and putting together full pictures.
 
OK, so I’m going to pull back the curtain and give you the rest of our process. First, this is just our program. Second, don’t hassle me if you think this is a stupid process. Third, I’m really not that anal, I just like to have data to use when a committee gets together to rank, even if the data is BS.:)

The overarching question we keep in mind when we rank our candidates is "How proud will we be of this trainee?". The sub-questions our selection committee has been asked to consider are the following: “Will we enjoy working with this person, can we trust them to do the best job possible for our patients, can we be confident in their ability to interact with our clinical colleagues, will this person help us recruit great residents in the future, will we be excited to see this person at a meeting after they graduate, are they on a career trajectory that could inspire future residents and maintain our brand?”
What does this mean? It might differ a little for each interviewer, but here’s a translation:

• Enjoy working with: obvious
• Trust them to do the best job: potential for developing excellent clinical skills, demeanor, motivation
• Interactions with clinical colleagues: If the most arrogant surgeon comes down to review a case, will this person be able to handle it well?
• Help us recruit: are they likeable, present themselves well, will they pursue the best possible fellowships
• Excited to see: obvious
• Inspiring career trajectory: Leadership (private practice, organized radiology, or academics), Academics, Community service.

In order to help us “guess” on the answers to these questions, we try to assess some basic traits in the interview. Each interviewer is asked to provide a score from 1-10 on the following items. Note that the interviewer is asked to try to glean this from the application (looking at letters of recommendation, Dean’s letter comments, CV/experiences/research record) as well as through any questions. We don’t use a behavioral interview question process per se (some of our interviewers use to an extent, others don’t), but the task of the interviewer is to try to use the interview to augment their assessment of the application to provide a score for each item. We try to assess these things, but I’ll be the first to admit that we are purely guessing on many of them—it’s all subjective. I am going to give you the exact wording from our evaluation form that interviewers have to score:
1. Personality & Energy: shows sense of humor, optimism; is adaptable/flexible, humble, calm, confident, likable, mature; displays social intelligence, social grace, empathy, creativity, buoyancy, curiosity, and is thorough; displays energy, passion, enthusiasm, excitement, interest, ‘‘zing’’
2. Pleaser: will do what is asked, won't complain, steady, "nose to the grindstone", motivated, inquisitive, grateful
3. Self-reliance: Exhibits evidence of self-improvement; reflective, solicits feedback; understands strengths/weaknesses; shows awareness of needs, learning, and self-assessment; able to "figure out what to do"; "go getter"; doesn't need a lot of direction
4. Interpersonal: Is this someone you want to work with daily? Is this person a team player? Are other residents going to enjoy this colleague?
5. Pure Smarts (generalized--not specific to one area); test taking skills (consider board scores, shelf exams, etc)
6. Research: consider not what they've done or how many publications but why they did it; can they speak to intent?; how likely are they to continue to do in residency; beyond residency?; consider goals
7. LORs/Dean's letter comments (from rotations): how outstanding are they? (5 is average)
8. Intangibles: Is anything bugging you about the candidate (if so, score less than 5) Is anything compelling about the candidate (if so, score higher than 5). Leave at 5 if nothing positive or negative; use this score to emphasize special concerns or highlight special skills

The sum of all scores becomes that interviewer’s total score. The median of these scores from all the interviewers for any one individual is put into the spreadsheet as the “Interview” score.

The scores from items 1, 2, 3, 4, and 8 are totaled and form this person’s “Personality” score. The average (not the median) Personality score is recorded on the spreadsheet.

The average of the score from item 6 is the person’s “Research” score. That is recorded on the spreadsheet. This has an influence on the "Leadership" assessment (see later).

The “Interview” score is combined with the “USMLE” score and the “Clinical” score (the 3rd year core clerkships) to determine the overall score. We constantly tweak the relative percentage that each score contributes to the overall score, but for the most part, the interview score is about 50%, the USMLE score is 25%, and the Clinical score is 25%.

At the end of the interview day, I enter the data into the spreadsheet. The residents who have met with the candidates are asked to send me a short note about anyone they have concerns about, and why…or anyone who really impressed them, and why. I put these in the person’s “comments” box. Note that the residents don’t have the person’s application materials.

The selection committee meets that day and we collectively review the spreadsheet (projected onto a big screen). We see how the numbers ranked the candidates by “overall score” and by “personality score”, but that is only a guide. We review any resident comments and determine if we collectively can “validate” any concerns—sometimes we don’t collectively agree and so we discount, other times it confirms a nagging suspicion. We collectively determine a “Leadership” score which tries to get at the “inspirational career trajectory” issue—this isn’t factored into the overall score, but it’s listed.

We end up ranking that day’s candidates from 1-100, even though we didn’t interview 100. This is a group discussion with lots of back and forth. We look at the individual scores (Personality, USMLE, Clinical, Research, Leadership). As I said, this relative ranking often does not follow the “overall score” number, but if two candidates are flip-flopped based on their numbers or someone seems underranked/overranked, we really scrutinize why. Did one interviewer just have a bad experience with one of the candidates, or really connect with a candidate, and is that skewing the data? Often times this helps us adjust the rank.

Again, the ranking of that day’s candidates is based on where we think that person might actually end up on our final rank list, based on our collective feelings about that candidate and memory of candidates in the past. So, if we interviewed 10 people, the ranks for that day might be 5-6-10-11-17-18-50-75-80-100. This means we found 2 candidates that look like top 10 material, 2 that were close to top 10 (maybe they will be, hard to say), 2 that were top 20% but probably not top 10, one middle of the road, 2 that are weak, 1 that we probably won’t rank.

Then we feather the candidates from the day into any existing ranks we have (from previous interview dates). This is the hardest part, because in general, I’m the only person who has interviewed every candidate—some of the selection committee members from that day will have interviewed some of the ones on other days, but not all. Here’s where we often will try to understand the interplay of the “Personality Score” vs. the “USMLE score” vs. the “Clinical score” for any one candidate compared to another. At the end of the day, a new set of ranks emerges that has feathered the new candidates to the ones from previous days. Again, there will be gaps between candidates that reflect the fact that we haven’t yet interviewed everyone. Sometimes, we keep two or three people at the same rank, meaning we have to revisit how to relatively rank that group at the end of the interview period.

At the end of all interviews, the entire selection committee goes through the spreadsheet. We keep the spreadsheet, the application packets, and each interview assessment in an electronic file available only to selection committee members, so that reviewers can go back to the raw data if needed. We then have a big final ranking meeting at the end where we take a 30,000 foot look to see if it makes sense--then, any “ties” are sorted out – individual committee members can also “advocate” for any candidates they want to move him/her up (and sometimes down) the ranks, and we use the numbers and evals as “evidence” during those discussions.

In the end, our “overall score” does not predict the exact ranking. Last year, our top 10 candidates (in order) had the following “overall score” ranks: 2-9-14-19-3-4-8-7-11-6. The number 1 person on the "overall score" ended up being ranked 11th.

So, there you have it!
 
  • Like
Reactions: 22 users
OK, so I’m going to pull back the curtain and give you the rest of our process. First, this is just our program. Second, don’t hassle me if you think this is a stupid process. Third, I’m really not that anal, I just like to have data to use when a committee gets together to rank, even if the data is BS.:)

The overarching question we keep in mind when we rank our candidates is "How proud will we be of this trainee?". The sub-questions our selection committee has been asked to consider are the following: “Will we enjoy working with this person, can we trust them to do the best job possible for our patients, can we be confident in their ability to interact with our clinical colleagues, will this person help us recruit great residents in the future, will we be excited to see this person at a meeting after they graduate, are they on a career trajectory that could inspire future residents and maintain our brand?”
What does this mean? It might differ a little for each interviewer, but here’s a translation:

• Enjoy working with: obvious
• Trust them to do the best job: potential for developing excellent clinical skills, demeanor, motivation
• Interactions with clinical colleagues: If the most arrogant surgeon comes down to review a case, will this person be able to handle it well?
• Help us recruit: are they likeable, present themselves well, will they pursue the best possible fellowships
• Excited to see: obvious
• Inspiring career trajectory: Leadership (private practice, organized radiology, or academics), Academics, Community service.

In order to help us “guess” on the answers to these questions, we try to assess some basic traits in the interview. Each interviewer is asked to provide a score from 1-10 on the following items. Note that the interviewer is asked to try to glean this from the application (looking at letters of recommendation, Dean’s letter comments, CV/experiences/research record) as well as through any questions. We don’t use a behavioral interview question process per se (some of our interviewers use to an extent, others don’t), but the task of the interviewer is to try to use the interview to augment their assessment of the application to provide a score for each item. We try to assess these things, but I’ll be the first to admit that we are purely guessing on many of them—it’s all subjective. I am going to give you the exact wording from our evaluation form that interviewers have to score:
1. Personality & Energy: shows sense of humor, optimism; is adaptable/flexible, humble, calm, confident, likable, mature; displays social intelligence, social grace, empathy, creativity, buoyancy, curiosity, and is thorough; displays energy, passion, enthusiasm, excitement, interest, ‘‘zing’’
2. Pleaser: will do what is asked, won't complain, steady, "nose to the grindstone", motivated, inquisitive, grateful
3. Self-reliance: Exhibits evidence of self-improvement; reflective, solicits feedback; understands strengths/weaknesses; shows awareness of needs, learning, and self-assessment; able to "figure out what to do"; "go getter"; doesn't need a lot of direction
4. Interpersonal: Is this someone you want to work with daily? Is this person a team player? Are other residents going to enjoy this colleague?
5. Pure Smarts (generalized--not specific to one area); test taking skills (consider board scores, shelf exams, etc)
6. Research: consider not what they've done or how many publications but why they did it; can they speak to intent?; how likely are they to continue to do in residency; beyond residency?; consider goals
7. LORs/Dean's letter comments (from rotations): how outstanding are they? (5 is average)
8. Intangibles: Is anything bugging you about the candidate (if so, score less than 5) Is anything compelling about the candidate (if so, score higher than 5). Leave at 5 if nothing positive or negative; use this score to emphasize special concerns or highlight special skills

The sum of all scores becomes that interviewer’s total score. The median of these scores from all the interviewers for any one individual is put into the spreadsheet as the “Interview” score.

The scores from items 1, 2, 3, 4, and 8 are totaled and form this person’s “Personality” score. The average (not the median) Personality score is recorded on the spreadsheet.

The average of the score from item 6 is the person’s “Research” score. That is recorded on the spreadsheet. This has an influence on the "Leadership" assessment (see later).

The “Interview” score is combined with the “USMLE” score and the “Clinical” score (the 3rd year core clerkships) to determine the overall score. We constantly tweak the relative percentage that each score contributes to the overall score, but for the most part, the interview score is about 50%, the USMLE score is 25%, and the Clinical score is 25%.

At the end of the interview day, I enter the data into the spreadsheet. The residents who have met with the candidates are asked to send me a short note about anyone they have concerns about, and why…or anyone who really impressed them, and why. I put these in the person’s “comments” box. Note that the residents don’t have the person’s application materials.

The selection committee meets that day and we collectively review the spreadsheet (projected onto a big screen). We see how the numbers ranked the candidates by “overall score” and by “personality score”, but that is only a guide. We review any resident comments and determine if we collectively can “validate” any concerns—sometimes we don’t collectively agree and so we discount, other times it confirms a nagging suspicion. We collectively determine a “Leadership” score which tries to get at the “inspirational career trajectory” issue—this isn’t factored into the overall score, but it’s listed.

We end up ranking that day’s candidates from 1-100, even though we didn’t interview 100. This is a group discussion with lots of back and forth. We look at the individual scores (Personality, USMLE, Clinical, Research, Leadership). As I said, this relative ranking often does not follow the “overall score” number, but if two candidates are flip-flopped based on their numbers or someone seems underranked/overranked, we really scrutinize why. Did one interviewer just have a bad experience with one of the candidates, or really connect with a candidate, and is that skewing the data? Often times this helps us adjust the rank.

Again, the ranking of that day’s candidates is based on where we think that person might actually end up on our final rank list, based on our collective feelings about that candidate and memory of candidates in the past. So, if we interviewed 10 people, the ranks for that day might be 5-6-10-11-17-18-50-75-80-100. This means we found 2 candidates that look like top 10 material, 2 that were close to top 10 (maybe they will be, hard to say), 2 that were top 20% but probably not top 10, one middle of the road, 2 that are weak, 1 that we probably won’t rank.

Then we feather the candidates from the day into any existing ranks we have (from previous interview dates). This is the hardest part, because in general, I’m the only person who has interviewed every candidate—some of the selection committee members from that day will have interviewed some of the ones on other days, but not all. Here’s where we often will try to understand the interplay of the “Personality Score” vs. the “USMLE score” vs. the “Clinical score” for any one candidate compared to another. At the end of the day, a new set of ranks emerges that has feathered the new candidates to the ones from previous days. Again, there will be gaps between candidates that reflect the fact that we haven’t yet interviewed everyone. Sometimes, we keep two or three people at the same rank, meaning we have to revisit how to relatively rank that group at the end of the interview period.

At the end of all interviews, the entire selection committee goes through the spreadsheet. We keep the spreadsheet, the application packets, and each interview assessment in an electronic file available only to selection committee members, so that reviewers can go back to the raw data if needed. We then have a big final ranking meeting at the end where we take a 30,000 foot look to see if it makes sense--then, any “ties” are sorted out – individual committee members can also “advocate” for any candidates they want to move him/her up (and sometimes down) the ranks, and we use the numbers and evals as “evidence” during those discussions.

In the end, our “overall score” does not predict the exact ranking. Last year, our top 10 candidates (in order) had the following “overall score” ranks: 2-9-14-19-3-4-8-7-11-6. The number 1 person on the "overall score" ended up being ranked 11th.

So, there you have it!

Thank you the amazing insight! I wish I had this earlier.

Now, I am interviewing for IR fellowships, where programs typically interview between 20-50 candidates. Do you think a similar process is involved or are ranking more based on gut feeling in general for fellowships?
 
Now, I am interviewing for IR fellowships, where programs typically interview between 20-50 candidates. Do you think a similar process is involved or are ranking more based on gut feeling in general for fellowships?


I have a little bit of a different perspective about residency selection, as a current applicant to IR.

I realized the most important aspect of medical training is to leave the MOST amount of doors open for the next stage of training. In general, the larger, more well known program you go to, the more doors remain open.

While many radiology PDs will tell you that "we always match people to Harvard or UCSF", that is both true and false in a sense. It's true because every year people will be able to get to UCSF or MGH from ALMOST any program.

What has left unsaid is that choices of residency really begin to matter when the competitiveness of a specialty begin to increase. It's simple, really, all being equal between two applicants, the one who went to a well known program will have more boxes checked.

I went to a small community radiology residency program, busted my asses during residency and did a lot of research as well as having excellent letters and good (240+) step 1 and 2. Meanwhile, a coresident came from a DO school and did no research due to having a lot of family responsibilities

He is applying to neuroradiology right now, and is deciding between big names like Harvard or Yale. Meanwhile, I am appying to IR and I did not get many big name invites. While I am incredibly happy with the fellowship programs I am getting and hopefully will match, once a subspecialty is competitive, it absolutely matters to where you go for residency because your letters will come from big names in the field PDs know and trust.

Plus, you never know where life will take you. You may decide you suddenly want to do IR. You may meet a woman who is hellbent to go back to live in NYC where she grew up.

So if I was back in my shoes during med school days, I would strongly consider going to where the program has the strongest name. Many programs provide very strong training but those with good names are a lot fewer.

Name will carry you to places and leave the most amount of door open.

Having been a fellowship PD in the past, I can say that it is very unlikely fellowships use this sort of process. We didn't. Yes, we tried to assess our impressions of candidates' "personalities" and who we wanted to work with day in and day out. We also looked at strength/reputation of residency program. LORs important. Personal connections to letter writers is huge. USMLE no longer really in play. Same with general med school performance. Career trajectory important, so research important. Part of your job as a fellow (and resident to some degree) is to help your attendings publish, since that can be important to their career--this is why demonstrated ability to get things done on the research side matters.
 
  • Like
Reactions: 1 users
Having been a fellowship PD in the past, I can say that it is very unlikely fellowships use this sort of process. We didn't. Yes, we tried to assess our impressions of candidates' "personalities" and who we wanted to work with day in and day out. We also looked at strength/reputation of residency program. LORs important. Personal connections to letter writers is huge. USMLE no longer really in play. Same with general med school performance. Career trajectory important, so research important. Part of your job as a fellow (and resident to some degree) is to help your attendings publish, since that can be important to their career--this is why demonstrated ability to get things done on the research side matters.

1. What would an average resident hope to produce, on average, in terms of publishings and/or presentations during the course of residency?

2. Same question applied to fellows/fellowship.

Thank you!
 
Having been a fellowship PD in the past, I can say that it is very unlikely fellowships use this sort of process. We didn't. Yes, we tried to assess our impressions of candidates' "personalities" and who we wanted to work with day in and day out. We also looked at strength/reputation of residency program. LORs important. Personal connections to letter writers is huge. USMLE no longer really in play. Same with general med school performance. Career trajectory important, so research important. Part of your job as a fellow (and resident to some degree) is to help your attendings publish, since that can be important to their career--this is why demonstrated ability to get things done on the research side matters.

Appreciate the insightful comment!
 
  • Like
Reactions: 1 user
Edit: and when you send the holy grail "you are ranked to match" email, what exactly does that mean?

We don't send out "rank to match" communications. I imagine, however, that for those programs that do, they are real (unless, somehow you are getting punked by a malevolent colleague). That is, if a program has 7 spots, they may have decided to send 7 rank to match communications.

I personally have never understood this and don't get why anyone (applicant or program) would change their rank list based on whether or not there was some understanding that the applicant was ranking the program #1 or vice versa. I can see why applicants might cancel interviews after getting one or two of these from places in which they are highly interested--it's validation that the applicant is very competitive. But no reason to rank a place higher because you got the "rank to match" letter, and no reason to rank a candidate higher because you got the "You're my number 1" letter. IMO.
 
  • Like
Reactions: 2 users
We don't send out "rank to match" communications. I imagine, however, that for those programs that do, they are real (unless, somehow you are getting punked by a malevolent colleague). That is, if a program has 7 spots, they may have decided to send 7 rank to match communications.

I personally have never understood this and don't get why anyone (applicant or program) would change their rank list based on whether or not there was some understanding that the applicant was ranking the program #1 or vice versa. I can see why applicants might cancel interviews after getting one or two of these from places in which they are highly interested--it's validation that the applicant is very competitive. But no reason to rank a place higher because you got the "rank to match" letter, and no reason to rank a candidate higher because you got the "You're my number 1" letter. IMO.

Is it not true that PDs are judged by the Chair by how far they go down their rank list? If that's the case, then there would be incentive to modify the list. Perhaps this is just one of those rumors.


Sent from my iPhone using SDN mobile
 
Again, I can't speak for everyone, and there are some crazy Chairs out there. I've never seen that. I guess those PDs should only invite less competitive applicants...obviously that doesn't happen.

It also presumes that applicants would actually rank a place higher because of an RTM communication. If, after your interviews, you decide Program A is better for you than Program B, you shouldn't really change your rank because Program B sends you a love note.
 
  • Like
Reactions: 2 users
Again, I can't speak for everyone, and there are some crazy Chairs out there. I've never seen that. I guess those PDs should only invite less competitive applicants...obviously that doesn't happen.

It also presumes that applicants would actually rank a place higher because of an RTM communication. If, after your interviews, you decide Program A is better for you than Program B, you shouldn't really change your rank because Program B sends you a love note.

I agree with you, of course. With how the algorithm works, it makes no sense for an applicant to rank B over A. But I always thought that applicants had an incentive to send programs the "you're my number 1" email.


Sent from my iPhone using SDN mobile
 
The amount of info and depth to these answers is astounding. Really great resource, thank you again for this thread.
 
  • Like
Reactions: 1 user
Top